You are on page 1of 3

DAILY ACTIVITY 1

5. For the figure


ADDITION, SUBTRACTION of
  
VECTORS, RESOLUTION of VECTORS (a) A  B  C
and RECTANGULAR COMPONENT    
(b) B  C  A C 
1. There are two force vectors, one of 5 N and B
  
(c) C  A  B
other of 12 N at what angle the two 
   A
vectors be added to get resultant vector (d) A  B  C  0
of 17 N, 7 N and 13 N respectively
6. When three forces of 50 N, 30 N and 15 N
(a) 0°, 180° and 90° (b) 0°, 90° and 180°
act on a body, then the body is
(c) 0°, 90° and 90° (d) 180°, 0° and 90°
(a) At rest
2. If the sum of two unit vectors is a unit
(b) Moving with a uniform velocity
vector, then magnitude of difference is
(c) In equilibrium
(a) 2 (b) 3
(d) Moving with an acceleration
(c) 1 / 2 (d) 5
7. The resultant of two vectors A and B is
3. A particle is simultaneously acted by two perpendicular to the vector A and its
forces equal to 4 N and 3 N. The net magnitude is equal to half the magnitude
force on the particle is of vector B. The angle between A and B is
(a) 7 N (a) 120° (b) 150°
(b) 5 N (c) 135° (d) None of these
(c) 1 N 8. If vectors P, Q and R have magnitude 5, 12
  
(d) Between 1 N and 7 N and 13 units and P  Q  R , the angle

4. If the resultant of the two forces has a between Q and R is


magnitude smaller than the (a) cos 1 5/12 (b) cos 1 5/ 13
magnitude of larger force, the two
forces must be (c) cos 1 12/13 (d) cos 1 7 / 13

(a) Different both in magnitude and 


9. What is the angle between P and the
direction    
resultant of ( P  Q ) and ( P  Q )
(b) Mutually perpendicular to one another
(a) Zero (b) tan 1 ( P / Q )
(c) Possess extremely small magnitude
(c) tan 1 (Q / P ) (d) tan 1 ( P  Q )/( P  Q)
(d) Point in opposite directions
 
10. Resultant of P and Q is perpendicular SOLUTION
  
to P . What is the angle between P & Q
1. (a) For 17 N both the vector should be
(a) cos 1  P / Q  (b) cos 1  P / Q  parallel i.e. angle between them should be zero.
For 7 N both the vectors should be antiparallel
(c) sin 1  P / Q  (d) sin 1  P / Q 
i.e. angle between them should be 180°
    
11. Given that A  B  C and that C is  to A .
For 13 N both the vectors should be
 
Further if | A | | C |, then what is the angle perpendicular to each other i.e. angle between
 
between A and B them should be 90°

(a)  / 4 radian (b)  / 2 radian


2. (b) Let n̂1 and n̂2 are the two unit vectors,
(c) 3  / 4 radian (d)  radian then the sum is

12. A scooter going due east at 10 ms–1 turns n s  nˆ 1  nˆ 2  n s2  n12  n 22  2n1n 2 cos   1  1  2 cos 

right through an angle of 90°. If the speed Since it is given that n s is also a unit vector,
of the scooter remains unchanged in 1
therefore 1  1  1  2 cos   cos       120
taking turn, the change is the velocity of 2

the scooter is Now the difference vector is nˆ d  nˆ 1  nˆ 2

(a) 20.0 ms–1 south eastern direction  n d2  n12  n 22  2n1n 2 cos   1  1  2cos(120)

(b) Zero  nd2  2  2 1 / 2  2  1  3  n d  3

(c) 10.0 ms–1 in southern direction  


3. (d) If two vectors A and B are given then
(d) 14.14 ms–1 in south-west direction
the resultant R max = A  B  7N and Rmin  4  3 1N
13. Which pair of the following forces will i.e. net force on the particle is between 1 N & 7 N.
never give resultant force of 2 N

(a) 2 N and 2 N (b) 1 N and 1 N 4. (d)

(c) 1 N and 3 N (d) 1 N and 4 N 5. (c)


14. Y-component of velocity is 20 and
6. (d) Here all the three force will not keep
X-component of velocity is 10. The
the particle in equilibrium so the net force will
direction of motion of the body with the
not be zero and the particle will move with
horizontal at this instant is
acceleration.
(a) tan 1
 2 (b) tan 1
 1 / 2
B
(c) 45° (d) 0° 7. (b)  A2  B2  2AB cos  …(i)
2
DAILY ACTIVITY 3

 tan 90 
Bsin 
 A  Bcos   0  cos   
A  cos   1 / 2    135  3 / 4 0

A  Bcos  B
N
B2 B 12. (d)
Hence, from (i)  A 2  B2  2A 2  A  3 
–v1

v1
4 2  W E

A 3 v

 cos        150 
v2
S
B 2
If the magnitude of vector remains same, only
     
8. (c) | P |  5 , | Q |  12 and | R |  13   direction change by  then v  v2  v1 ,
R 
  
Q
v  v2  ( v1 )
   cos 1  
Q 12 12
 cos   
R 13  13    
 Magnitude of change in vector | v |  2v sin  
P 2  
     
9. (a) Resultant R  P  Q  P  Q  2P
  90 
  | v |  2  10  sin    10 2  14.14 m / s
The angle between P and 2P is zero.  2 
Direction is south-west as shown in figure.
10. (b) R
Q
13. (d) If two vectors A and B are given then
Range of their resultant can be written as

P (A  B)  R  (A  B) .

Qsin  i.e. R max  A  B and R min  A  B


 tan 90   P  Q cos   0
P  Qcos 
If B = 1 and A = 4 then their resultant will lies in
P  P  between 3N and 5N. It can never be 2N.
 cos      cos 1  
Q  Q 

    14. (a) Given: v y  20 and v x  10


11. (c)Given: C  A and | A || C |

       Velocity  v   10iˆ  20ˆj
Given: A  B  C  B  C  A
vy 20
Direction of velocity with x axis: tan    2
vx 10
  B    C    A   2 AB cos900
2 2 2
y

   tan 1 (2)
  B    A   A  2  A
2 2 2 2
 BA 2 v
vy

 
For angle between A and B
 x
   vx
A  B  C   C    A    B   2 AB cos
2 2 2

   
2
  A   A  A 2
2 2
 2 A A 2 cos

You might also like